LSAT and Law School Admissions Forum

Get expert LSAT preparation and law school admissions advice from PowerScore Test Preparation.

User avatar
 Dave Killoran
PowerScore Staff
  • PowerScore Staff
  • Posts: 5853
  • Joined: Mar 25, 2011
|
#88526
Complete Question Explanation
(The complete setup for this game can be found here: lsat/viewtopic.php?f=181&p=88524#p88524)

The correct answer choice is (D).

If H is given on the second day of the training conference, then the only remaining space T can occupy is the 2nd seminar on the third day (T cannot be given on the first day due to the second rule, and if H is on the second day, then T cannot be on the second day due to the first rule. Thus, T must be given on the third day).

With H given on the second day, and T given on the third day, the remaining long seminar—N—must be given on the first day. Once N is given on the first day, the third rule is enacted, and P must also be given on the first day (as the 1st seminar). This information results in the following setup:

G3-Q17-d1.png

The only two remaining variables are G and O. Because T is already the final seminar given (and thus the second rule is satisfied), G and O are split between the second and third days: one of G and O is on the second day, and one of G and O is on the third day (as the 1st seminar). Hence, O can be given on the third day, and answer choice (D) is correct.
You do not have the required permissions to view the files attached to this post.

Get the most out of your LSAT Prep Plus subscription.

Analyze and track your performance with our Testing and Analytics Package.